LSAT and Law School Admissions Forum

Get expert LSAT preparation and law school admissions advice from PowerScore Test Preparation.

 Administrator
PowerScore Staff
  • PowerScore Staff
  • Posts: 8916
  • Joined: Feb 02, 2011
|
#23347
Complete Question Explanation

Parallel Flaw. The correct answer choice is (E)

The stimulus of this problem contains a flawed reasoning because it is missing the basis for comparison:
  • 75% of strict vegetarians reach 50 without developing heart disease
Sub-conclusion:
  • Cause ..... ..... ..... ..... Effect

    Vegetarianism ..... :arrow: ..... Avoid Heart Disease
Therefore, to reduce risk of heart disease, one should not eat meat

This is, however, incorrect reasoning since we do not know what percentage of non-vegetarians reach 50 without developing heart disease. If 80% of non-vegetarians reach 50 without developing heart disease, for example, then the sub-conclusion of this argument is no longer valid, neither is the final conclusion / recommendation. Thus it is an incorrect reasoning.

Answer choice (A): The reasoning in this answer choice is sound, so this cannot be the right answer choice. The conclusion of this argument also does not match that of the stimulus: it should say something to the effect of "To avoid traffic accidents, one should drive below the speed limit". Finally, this argument is missing a sub-conclusion, which the stimulus has.

Answer choice (B): The reasoning in this answer choice is sound as well. It is not missing the basis for comparison: it states that cigarette smokes have a greater chance of heart disease than non-smokers. Also, this answer choice is missing a sub-conclusion, which the stimulus has.

Answer choice (C): This answer is missing a subsidiary conclusion, which the stimulus has. Moreover, the conclusion of this answer also does not match that of the stimulus: it should conclude, instead, that "people who want to avoid dental problems should not drink coffee".

Answer choice (D): This answer choice is not missing the basis for comparison because it states that "people who do not exercise regularly have a shorter life expectancy than those who exercise regularly". Moreover, this choice lacks a sub-conclusion. Finally, the conclusion does not match that of the stimulus: it should instead state that "people who wish to live a longer life should exercise regularly."

Answer choice (E): This is the correct answer choice.
  • Most people who exercise regularly can handle stress
Sub-conclusion:
  • Cause ..... ..... ..... ..... ..... Effect

    Exercising Regularly ..... :arrow: ..... Decrease Chance of Being Overwhelmed by Stress
Thus, to handle stress better, one should exercise regularly

Again, this is an incorrect reasoning since it is missing the basis for comparison: if everyone who does not exercise regularly can handle stress, for example, then it seems that exercising regularly is actually bad for handling stress, and the sub-conclusion and conclusion of this argument will no longer stand. All the parts of this argument match those of the stimulus, and it commits the same flaw, thus it is the correct answer.
 brettb
  • Posts: 14
  • Joined: Mar 29, 2016
|
#23187
I got this question right and would like to confirm that my technique is correct.

The argument states that 75% strict vegetarians reached age 50 without developing heart disease (WDHD).
I interpreted this as Majority Strict Vegetarians (Avoid Meat) reached age 50 WDHD.

The subsidiary conclusion says Avoiding Meat Increases one chance of avoiding DHD.
I interpreted this as Everyone Who Avoids Meat increases chance of avoiding DHD

The Flaw is taking something that is true to a majority and applying it to all.

Another thing I noted to parallel was in the conclusion, "People who want to do something should take that action them self".

A) Brings in new information (police officers) - Eliminate
B) I was skeptical of this because it dealt with heart disease just like this stimulus. But eliminated because it did not make the shift from a Majority > All.
C) Starts off with Majority which is what I'm looking for. Also has correlation which seems relevant given the statistics. But never makes the shift I was specifically looking for. I was able to confirm elimination when it said the government should cause the change, not the person.
D) Never mentions anything about a majority - Eliminate.
E) Most people (majority). This shows that exercising (Implies all) Makes shift I'm looking for. Lastly, People who want to handle stress better make change them self. Bingo!

If someone could tell me if there was anything wrong with why I eliminated an answer choice that would be great. Specifically C.

Thanks!
 Claire Horan
PowerScore Staff
  • PowerScore Staff
  • Posts: 408
  • Joined: Apr 18, 2016
|
#23236
Hi Brett,

Your reasons for eliminating C are fine. I would add what I think is the biggest reason C is different. C seems much more reasonable, and here we are looking for a flawed argument. The reason C is a more reasonable argument is that it considers a majority of a group that behaves in a particular way and later experiences a problem. This is a stronger reason to stop doing the behavior than in the stimulus, where the majority of the group members abstain from a behavior and do not suffer a consequence. There is no control suggesting that of a similar group of meat eaters, less than 75% were able to avoid developing heart disease before age 50. E is similar in that it is about a group of people who do not suffer a consequence.

-Claire
LSAT and GRE Instructor
 cpando1995@gmail.com
  • Posts: 18
  • Joined: Jul 07, 2016
|
#27703
Hi,

Is there also a Mistaken Reversal in the stimulus for this question? I diagrammed the stimulus the following way:

P: ~Meat --> ~Serious heart disease

C: Reduced risk of heart disease (~Serious heart disease) --> ~Meat


Then, I recognized that answer choice E was the only one that had this same flaw and that's how I arrived at that answer choice. I diagrammed answer choice E as follows:

P: Exercise regularly-->Handle stress

C: Handle stress--> Exercise regularly


Did I approach this question the right way, or did I just happen to get the right answer?

Thank you
 David Boyle
PowerScore Staff
  • PowerScore Staff
  • Posts: 836
  • Joined: Jun 07, 2013
|
#27735
cpando1995@gmail.com wrote:Hi,

Is there also a Mistaken Reversal in the stimulus for this question? I diagrammed the stimulus the following way:

P: ~Meat --> ~Serious heart disease

C: Reduced risk of heart disease (~Serious heart disease) --> ~Meat


Then, I recognized that answer choice E was the only one that had this same flaw and that's how I arrived at that answer choice. I diagrammed answer choice E as follows:

P: Exercise regularly-->Handle stress

C: Handle stress--> Exercise regularly


Did I approach this question the right way, or did I just happen to get the right answer?

Thank you

Hello cpando1995@gmail.com,

Your diagrams don't happen to include the idea of "most", so may not be too close to what the stimulus says. Also, arrows usually point to something "necessary" or "required", but that's not exactly the idea here, since meatlessness and exercise are supposed to be helpful, which is not identical to being necessary.

Hope this helps,
David
 Johnclem
  • Posts: 122
  • Joined: Dec 31, 2015
|
#28597
hello,
Just a quick question .
I usually read parallel arguments for structure. The second I see the conclusion doesn't match I toss out the answer, so that's want I did here . Our conclusion in the stimulus says we should not ... Only B matched for this .

Should I quit this Practice ? Or am I missing something in the conclusion of the correct answer ?


Thanks
John
 Emily Haney-Caron
PowerScore Staff
  • PowerScore Staff
  • Posts: 577
  • Joined: Jan 12, 2012
|
#28649
Hi John,

Yeah, being so rigid in applying that strategy is a recipe for disaster, because you're looking to match the flaw in the reasoning, which may not match the structure or language. Prephrasing for yourself what the flaw is will help you find the right answer choice more successfully.
 jessicamorehead
  • Posts: 84
  • Joined: Jul 07, 2017
|
#39066
Is my logic okay on this one? I didn't originally see the flaw, so I resorted to comparing the structures of the arguments.

A) out because doesn't include an inter. con. that specifies a probability "aka increases one chances"
B) out because same topic of heart disease
C) out because discusses correlation, which is not mentioned in the main one
D) out because the first sentence doesn't offer any sign of "majority/most/75%/etc."
E) only one left. First sentence has the "most". Second sentence has the probabilistic component. Conclusion has the same structure of "people who want to A should do B"

If I'm completely stuck and don't see the flaw in the stimulus, is it okay to base my answer choice off of similarities with the original argument?
 AthenaDalton
PowerScore Staff
  • PowerScore Staff
  • Posts: 296
  • Joined: May 02, 2017
|
#39364
Hi Jessica,

Yes, looking for similarly-structured arguments is an acceptable way to tackle parallel reasoning questions. Your reasons for eliminating the answer choices are mostly on-point, but I will point out that answer choice (B) can be eliminated based on the fact that its reasoning is actually sound, unlike the reasoning in the stimulus.

The flaw in the reasoning here is that the stimulus fails to provide any comparison group to vegetarians. The stimulus says that 75 percent of vegetarians reach the age of 50 without developing heart disease, therefore eating a vegetarian diet must protect against heart disease.

The problem with this line of argument is that we have no idea how many carnivores reach the age of 50 without developing heart disease. If 75 percent of carnivores also reach this age without heart disease, then it would appear that being a vegetarian has absolutely no effect on heart disease. Alternately, if only 60 percent of carnivores reach the age of 50 without heart disease, the author's argument that a vegetarian diet is heart-healthy starts to look more credible. Either way, it's impossible to assess the argument without knowing how non-vegetarians' health compares to vegetarians' health.

Thanks for your question, and best of luck on the test!

Athena Dalton
 student987
  • Posts: 28
  • Joined: Apr 09, 2018
|
#49190
Hi. I noticed that there's an intermediary conclusion in the stimulus, and I went to (E) because that's the only one with an intermediary conclusion among the answer choices. Is this technique all right to use?

Get the most out of your LSAT Prep Plus subscription.

Analyze and track your performance with our Testing and Analytics Package.